Skip to main content

The Lebesgue Integral

  • Chapter
  • First Online:
Real Analysis

Part of the book series: Birkhäuser Advanced Texts Basler Lehrbücher ((BAT))

  • 4747 Accesses

Abstract

Let \(\{X,\mathcal {A},\mu \} \) be a measure space and \(E\in \mathcal {A}\).

This is a preview of subscription content, log in via an institution to check access.

Access this chapter

Chapter
USD 29.95
Price excludes VAT (USA)
  • Available as PDF
  • Read on any device
  • Instant download
  • Own it forever
eBook
USD 39.99
Price excludes VAT (USA)
  • Available as EPUB and PDF
  • Read on any device
  • Instant download
  • Own it forever
Softcover Book
USD 49.99
Price excludes VAT (USA)
  • Compact, lightweight edition
  • Dispatched in 3 to 5 business days
  • Free shipping worldwide - see info
Hardcover Book
USD 49.99
Price excludes VAT (USA)
  • Durable hardcover edition
  • Dispatched in 3 to 5 business days
  • Free shipping worldwide - see info

Tax calculation will be finalised at checkout

Purchases are for personal use only

Institutional subscriptions

Notes

  1. 1.

    Although referred to as the Radon-Nikodým theorem, the first version of this theorem, in the context of a measure in \(\mathbb {R}^N\) absolutely continuous with respect to the Lebesgue measure, is in Lebesgue [90]. Radon extended it to Radon measures in [121], and Nikodým to general measures in [115, 116].

Author information

Authors and Affiliations

Authors

Corresponding author

Correspondence to Emmanuele DiBenedetto .

Appendices

Problems and Complements

1c Measurable Functions

In the problems 1.1–1.12 \(\{X,\mathcal {A},\mu \} \) is a measure space and \(E\in \mathcal {A}\).

1.1. :

The characteristic function of a set E is measurable if and only if E is measurable.

1.2. :

A function f is measurable if and only if its restriction to any measurable subset of its domain is measurable.

1.3. :

A function f is measurable if and only if \(f^+\) and \(f^-\) are both measurable.

1.4. :

Let \(\{X,\mathcal {A},\mu \} \) be complete. A function defined on a set of measure zero is measurable.

1.5. :

Let \(\{X,\mathcal {A},\mu \} \) not be complete. Then there exists a measurable set A of measure zero that contains a nonmeasurable set B. The two functions \(f=\chi _A\) and \(g=\chi _{A-B}\) differ on a set of outer measure zero. However f is measurable and g is not.

1.6. :

If f is measurable then \([f=c]\) is measurable for all c in the range of f. The converse is false.

1.7. :

Let f be measurable. Then also \(|f|^{p-1}f\) is measurable for all \(p>0\).

1.8. :

|f| measurable does not imply that f is measurable. Likewise \(f^2\) measurable does not imply that f is measurable.

1.9. :

A function f is measurable if and only if its preimage of a Borel set is measurable.

1.10. :

Let \(\{f_n\}\) be a sequence of measurable functions from E into \(\mathbb {R}^*\). The subset of E where \(\lim f_n\) exists is measurable.

1.11. :

The supremum(infimum) of an uncountable family \(\{f_\alpha \}\) of measurable functions, need not be measurable.

1.12. :

Upper(lower) semi-continuous functions \(f:E\rightarrow \mathbb {R}^*\) are measurable.

In the problems 1.13–1.19, \(\{\mathbb {R}^N,\mathcal {M},\mu \}\) is \(\mathbb {R}^N\) with the Lebesgue measure and \(E\in \mathcal {M}\).  

1.13. :

There exists a nonmeasurable function \(f:\mathbb {R}\rightarrow \mathbb {R}\) such that \(f^{-1}(y)\) is measurable for all \(y\in \mathbb {R}\). Hint: given a nonmeasurable set \(E\subset \mathbb {R}\), define \(f(x)=x\) for \(x\in \mathbb {R}-E\) and \(f(x)=-x\) for \(x\in E\).

1.14. :

A monotone function f in some interval \((a,b)\subset \mathbb {R}\) is measurable.

1.15. :

Let \(f:\mathbb {R}\rightarrow \mathbb {R}\) be measurable and let \(g:\mathbb {R}\rightarrow \mathbb {R}\) be continuous. The composition \(g(f):E\rightarrow \mathbb {R}\) is measurable. However the composition \(f(g):E\rightarrow \mathbb {R}\) in general is not measurable. To construct a counterexample consider the function of § 14 of Chap. 3.

1.16. :

Let \(f:[0,1]\rightarrow \mathbb {R}^*\) be measurable. Then \(\chi _{\mathbf {Q}\cap [0,1]}(f)\), is measurable.

1.17. :

Let \(f:\mathbb {R}^2\rightarrow \mathbb {R}^*\) be such that \(f(\cdot ,y)\) is continuous for all \(y\in \mathbb {R}\) and \(f(x,\cdot )\) is measurable for all \(x\in \mathbb {R}\). Prove that f is measurable.

We indicate two approaches to this statement. The first is based on the following lemma.

 

Lemma 1.1c

Let \(c\in \mathbb {R}\) and let \(y\in \mathbb {R}\) be fixed. Then \(f(x,y)\ge c\) if and only if, for all \(n\in \mathbb {N}\) there exists a rational number \(r_m\) such that

$$\begin{aligned} |x-r_m|<{\textstyle \frac{1}{n}}\qquad \text { and }\qquad f(r_m,y)>c-{\textstyle \frac{1}{n}}. \end{aligned}$$

Proof

Since \(g=f(\cdot ,y)\) is continuous \(g^{-1}(c-\frac{1}{n},\infty )\) is open and we may select \(r_m\in \mathbb {Q}\) with the indicated properties. Conversely, let x satisfy the property of the lemma, and assume by contradiction that \(g(x)<c\). Since g is continuous, there exists \(k\in \mathbb {N}\) such that \(g(x)<c-\frac{1}{k}\). Since \(x\in g^{-1}(-\infty ,c-\frac{1}{k})\), and since this set is open, there exists \(h\in \mathbb {N}\) such that

$$\begin{aligned} (x-{\textstyle \frac{1}{h}},x+{\textstyle \frac{1}{h}}) \subset g^{-1}(-\infty ,c-{\textstyle \frac{1}{k}}). \end{aligned}$$

Take \(n=\max \{k;h\}\) and select any \(r\in \mathbb {Q}\) such that

$$\begin{aligned} |x-r|<{\textstyle \frac{1}{m}}\quad \text { and }\quad r\in g^{-1}(-\infty ,c-{\textstyle \frac{1}{k}}). \end{aligned}$$

For such choices

$$\begin{aligned} g(r)=f(r,y)<c-{\textstyle \frac{1}{n}}\le c-{\textstyle \frac{1}{k}}. \end{aligned}$$

\(\blacksquare \)

Using the lemma

$$\begin{aligned}{}[f\ge c]=\mathop {\textstyle {\bigcap }}\limits _n\mathop {\textstyle {\bigcup }}\limits _m\Big ( \big [f(r_m,\cdot )>c-{\textstyle \frac{1}{m}}\big ]\times B_{\frac{1}{n}}(r_m)\Big ). \end{aligned}$$

The second approach is based on the following construction. For \(n\in \mathbb {N}\) and \(j\in \mathbb {Z}\) set \(a_i=i/n\) and

$$\begin{aligned} f_n(x,\cdot )=\frac{f(a_{j+1},\cdot )(x-a_j) -f(a_j,\cdot )(x-a_{j+1})}{a_{j+1}-a_j},\> \quad \text { for }\> a_{j+1}\le x\le a_j. \end{aligned}$$

 

1.18. :

Find an example of a function \(f:\mathbb {R}^2\rightarrow \mathbb {R}^*\) measurable in each of its variables separately, and not measurable. Hint: see the Sierpinsky example 13.4. of § 13c.

1.19. :

Let T be a linear nonsingular transformation of \(\mathbb {R}^N\) onto itself and let \(f:\mathbb {R}^N\rightarrow \mathbb {R}^*\) be measurable. Prove that \(f(T):\mathbb {R}^N\rightarrow \mathbb {R}^*\) is measurable. Hint: By § 12.3c of Chap. 3 it suffices to show that

$$\begin{aligned}{}[f(T)>c]=T^{-1}[f>c]\quad \text { for all }\> c\in \mathbb {R}. \end{aligned}$$

 

1.1 1.1c Sublevel Sets

Let \(\{X,\mathcal {A},\mu \} \) be a measure space and let \(f:X\rightarrow \mathbb {R}\) be measurable. For \(t\in \mathbb {R}\), the sublevel set of f at t, is defined as \(E_t=[f\le t]\). By the definition

$$\begin{aligned} E_s\subset E_t \>\text { for }\> s<t;\quad \mathop {\textstyle {\bigcup }}\limits E_t=X;\quad \mathop {\textstyle {\bigcap }}\limits E_t=\emptyset ;\quad \mathop {\textstyle {\bigcap }}\limits _{s<t} E_t=E_s. \end{aligned}$$
(1.1c)

Conversely, given a collection of measurable sets \(\{E_t\}_{t\in \mathbb {R}}\) in \(\varOmega \), satisfying (1.1c) there exists a measurable function \(f:X\rightarrow \mathbb {R}\) for which \(E_t=[f\le t]\). Verify that (von Neumann [114])

$$\begin{aligned} X\ni x\rightarrow f(x)=\inf \{t:x\in E_t\} \end{aligned}$$

is such a function and prove that it is unique.

2c The Egorov–Severini Theorem

 

2.1. :

Let \(\{X,\mathcal {A},\mu \} \) be a measure space and let \(E\in \mathcal {A}\) be of finite measure. Let \(\{f_n\}\) be a sequence of measurable functions from E into \(\mathbb {R}^*\). Assume that for a.e. \(x\in E\) the set \(\{f_n(x)\}\) is bounded. For every \(\varepsilon >0\) there exists a measurable set \(E_\varepsilon \subset E\) and a positive number \(k_\varepsilon \) such that

$$\begin{aligned} \mu (E-E_\varepsilon )\le \varepsilon \quad \text { and }\> |f_n|\le k_\varepsilon \quad \text { on }\> E_\varepsilon \quad \text { for all }\> n\in \mathbb {N}. \end{aligned}$$
2.2. :

Let \(\mu \) be the counting measure on \(2^{\mathbb {N}}\). Define \(f_n\) as the characteristic function of \(\{1,\dots ,n\}\). The sequence \(\{f_n\}\) converges to 1 everywhere in \(\mathbb {N}\) but not in measure.

 

Proposition 2.1c

Let \(\{X,\mathcal {A},\mu \} \) be a measure space and let \(E\in \mathcal {A}\). Let \(\{f_n\}\) and f be measurable functions from E into \(\mathbb {R}^*\). Assume that f is finite a.e. in E. Then \(\{f_n\}\rightarrow f\) a.e. in E if and only if for all \(\eta >0\)

$$\begin{aligned} \lim \mu \big (\mathop {\textstyle {\bigcup }}\limits _{j=n}^\infty \left[ |f_j-f|\ge \eta \right] \big )=0. \end{aligned}$$
(2.1c)

Hint: Denoting by A the set where \(\{f_n\}\) is not convergent

$$\begin{aligned} A=\mathop {\textstyle {\bigcup }}\limits _m\limsup \big [|f_n-f|\ge {\textstyle \frac{1}{m}}\big ]. \end{aligned}$$

Then \(\mu (A)=0\) if and only if (2.1c) holds.

3c Approximating Measurable Functions by Simple Functions

Proposition 3.1c

Let \(\{X,\mathcal {A},\mu \} \) be a measure space and let \(f:X\rightarrow \mathbb {R}^*\) be nonnegative and measurable. There exists a countable collection \(\{E_j\}\subset \mathcal {A}\) such that \(X=\mathop {\textstyle {\bigcup }}\limits E_j\) and

$$\begin{aligned} f=\mathop {\textstyle {\sum }}\limits _{j\in \mathbb {N}} \frac{1}{j}\chi _{E_j}. \end{aligned}$$
(3.1c)

Proof

Let \(E_1=[f\ge 1]\) and \(f_1=\chi _{E_1}\). Then, for \(j\ge 2\) define recursively

$$\begin{aligned} E_j=\Big [f\ge \frac{1}{j}+f_{j-1}\Big ]\qquad \text { where }\qquad f_j=\mathop {\textstyle {\sum }}\limits _{i=1}^{j}\frac{1}{i}\chi _{E_i}. \end{aligned}$$

By construction \(f\ge f_j\) for all j. If \(f(x)=\infty \) then \(x\in E_j\) for all j. Hence f(x) has the representation (3.1c). If \(f(x)<\infty \) then \(x\notin E_j\) for infinitely many j. For such an x

$$\begin{aligned} 0\le f(x)-f_j(x)\le \frac{1}{j+1}\quad \text { for infinitely many}\, j. \end{aligned}$$

\(\blacksquare \)

The functions \(f_j\) are simple and hence the proposition gives an alternative proof of Proposition 3.1. However simple functions, even in canonical form, cannot be written, in general, as a finite sum of the type of (3.1c). As an example:  

3.1. :

Find the representation (3.1c) of a positive, real multiple of the characteristic function of the interval (0, 1).

3.2. :

Find the representation (3.1c) of a simple function taking only two positive values on distinct, measurable sets.

 

4c Convergence in Measure

In the problems 4.1–4.7, \(\{X,\mathcal {A},\mu \} \) is complete measure space, and \(E\in \mathcal {A}\) is of finite measure.  

4.1. :

Let \(f:E\rightarrow \mathbb {R}^*\) be measurable and assume that \(|f|>0\) a.e. on E. For every \(\varepsilon >0\) there exists a measurable set \(E_\varepsilon \subset E\) and a positive number \(\delta _\varepsilon \) such that \(\mu (E-E_\varepsilon )\le \varepsilon \) and \(|f|>\delta _\varepsilon \) on \(E_\varepsilon \).

4.2. :

Let \(\{f_n\}:E\rightarrow \mathbb {R}^*\) be a sequence of measurable functions converging to f a.e. in E. Assume that \(|f|>0\) and \(|f_n|>0\) a.e. on E for all \(n\in \mathbb {N}\). For every \(\varepsilon >0\) there exists a measurable set \(E_\varepsilon \subset E\) and a positive number \(\delta _\varepsilon \) such that

$$\begin{aligned} \mu (E-E_\varepsilon )\le \varepsilon \quad \text { and }\> |f_n|>\delta _\varepsilon \quad \text { on }\>E_\varepsilon \quad \text { for all }\>n\in \mathbb {N}. \end{aligned}$$
4.3. :

Let \(\mu \) be the counting measure on the rationals of [0, 1]. Then convergence in measure is equivalent to uniform convergence .

4.4. :

Let \(\{f_n\}:E\rightarrow \mathbb {R}^*\) be a sequence of measurable and a.e. finite functions. There exists a sequence of positive numbers \(\{k_n\}\) such that \(f_nk_n^{-1}\rightarrow 0\) a.e. in E.

4.5. :

Let \(\{f_n\},\,\{g_n\}: E\rightarrow \mathbb {R}^*\) be sequences of measurable functions converging in measure to f and g, respectively, and let \(\alpha ,\beta \in \mathbb {R}\). Then

$$\begin{aligned} \{\alpha f_n+\beta g_n\},\quad \{|f_n|\},\quad \{f_n g_n\}\quad \longrightarrow \quad \alpha f+\beta g,\quad |f|,\quad fg \end{aligned}$$

in measure. Moreover if \(f\ne 0\) a.e. on E and \(f_n\ne 0\) a.e. on E for all n, then \(1/f_n\) converges to 1 / f in measure. (Hint: Use 4.1–4.2).

4.6. :

Let \(\{X,\mathcal {A},\mu \} \) be a measure space and let \(\{E_n\}\) be a sequence of measurable sets. The sequence \(\{\chi _{E_n}\}\) converges in measure if and only if \(d(E_n;E_m)\rightarrow 0\) as \(n,m\rightarrow \infty \) (see 2.2. and 3.7. of the Complements of Chap. 3).

4.7. :

Let \(\{f_n\}:E\rightarrow \mathbb {R}^*\) be a sequence of measurable and a.e. finite functions. Prove that \(\{f_n\}\rightarrow f\) in measure if and only if every subsequence of \(\{f_n\}\) contains in turn a subsequence converging to f in measure.

 

7c The Lebesgue Integral of Nonnegative Measurable Functions

1.1 7.1c Comparing the Lebesgue Integral with the PeanoJordan Integral

Let \(E\subset \mathbb {R}^N\) be bounded and PeanoJordan measurable, and denote by \(\mathcal {P}=\{E_n\}\) a finite partition of E into pairwise disjoint PeanoJordan measurable sets. For a bounded function \(f:E\rightarrow \mathbb {R}\) set

$$\begin{aligned} \begin{array}{cc} h_n={\inf }_{x\in E_n}f(x)\qquad &{}\mathcal {F}^-_{\mathcal {P}}=\mathop {\textstyle {\sum }}\limits \,h_n \mu _{{}_{\mathcal {P}-\mathcal {J}}}(E_n)\\ k_n={\sup }_{x\in E_n}f(x)\qquad &{}\mathcal {F}^+_{\mathcal {P}}=\mathop {\textstyle {\sum }}\limits \,k_n \mu _{{}_{\mathcal {P}-\mathcal {J}}}(E_n). \end{array} \end{aligned}$$

A bounded function \(f:E\rightarrow \mathbb {R}\) is PeanoJordan integrable if for every \(\varepsilon >0\), there exists a partition \(\mathcal {P}_\varepsilon \) of E into PeanoJordan measurable sets \(E_n\), such that

$$\begin{aligned} \mathcal {F}^+_{\mathcal {P}}-\mathcal {F}^-_{\mathcal {P}}\le \varepsilon . \end{aligned}$$

The sets \(E_n\) are also Lebesgue measurable. Therefore,

$$\begin{aligned} \mathop {\textstyle {\sum }}\limits \,h_n\mu _{\mathcal {P}-\mathcal {J}}(E_n)\le \int _Efd\mu \le \mathop {\textstyle {\sum }}\limits \, k_n\mu _{\mathcal {P}-\mathcal {J}}(E_n). \end{aligned}$$

Thus if a bounded function f is PeanoJordan integrable it is also Lebesgue integrable. The converse is false. Indeed the characteristic function of the rationals \(\mathbb {Q}\) of [0, 1] is Lebesgue integrable and not PeanoJordan integrable.

This is not longer the case however if f is not bounded. Following Riemann’s notion of improper integral, the function

$$\begin{aligned} f(x)=\frac{1}{x}\,\sin \frac{1}{x}\qquad \text { for }\quad x\in (0,1] \end{aligned}$$

is PeanoJordan integrable in (0, 1) but not Lebesgue integrable.

In the problems 7.1–7.7, \(\{X,\mathcal {A},\mu \} \) is a measure space and \(E\in \mathcal {A}\).  

7.1. :

Let \(f:E\rightarrow \mathbb {R}^*\) be measurable and nonnegative. If \(f=\infty \) in a set \(\mathcal {E}\subset E\) of measure zero

$$\begin{aligned} \int _Ef d\mu =\int _{E-\mathcal {E}}fd\mu . \end{aligned}$$
7.2. :

Let \(f:X\rightarrow \mathbb {R}^*\) be measurable and nonnegative. Then

$$\begin{aligned} \int _Efd\mu =0\quad \text { for all }\> E\in \mathcal {A} \quad \text { implies }\quad f=0\> \text { a.e. in }\> X. \end{aligned}$$
7.3. :

Let \(f:E\rightarrow \mathbb {R}^*\) be integrable. If the integral of f over every measurable subset \(A\subset E\) is nonnegative, then \(f\ge 0\) a.e. on E.

7.4. :

Let \(f:\mathbb {R}\rightarrow \mathbb {R}\) be Lebesgue integrable. Then for every \(h\in \mathbb {R}\) and every interval \([a,b]\subset \mathbb {R}\)

$$\begin{aligned} \int _{[a,b]}fd\mu =\int _{[a+h,b+h]}f(x-h)d\mu . \end{aligned}$$
7.5. :

Construct the Lebesgue integral of a nonnegative \(\mu \)-measurable function \(f:\mathbb {R}^N\rightarrow \mathbb {R}\), when \(\mu \) is the Dirac delta measure \(\delta _x\) concentrated at some \(x\in \mathbb {R}^N\).

7.6. :

Let E be of finite measure. A measurable function \(f:E\rightarrow \mathbb {R}^*\) is integrable if and only if \(\mathop {\textstyle {\sum }}\limits \mu ([|f|\ge n])<\infty \). Hint:

$$\begin{aligned} \mathop {\textstyle {\sum }}\limits \mu ([|f|\ge n])=\mathop {\textstyle {\sum }}\limits n\mu ([n\le |f|<n+1]). \end{aligned}$$
7.7. :

Let \(\{X,\mathcal {A},\mu \} \) be \(\mathbb {R}^N\) with the Lebesgue measure. Let T be a linear nonsingular transformation of \(\mathbb {R}^N\) onto itself and let \(f:\mathbb {R}^N\rightarrow \mathbb {R}^*\) be Lebesgue integrable. Prove that

$$\begin{aligned} \int _Ef(x) d\mu =\frac{1}{|\det T|}\int _{TE} f(y) d\mu . \end{aligned}$$

Hint: § 12.3.1c of the Complements of Chap. 3 and Problem 1.19.

 

1.2 7.2c On the Definition of the Lebesgue Integral

The original definition of Lebesgue was based only on the Lebesgue measure in \(\mathbb {R}^N\). The integral of a measurable, nonnegative function f was defined as in (7.1), where the supremum was taken over the class of simple functions \(\zeta \le f\), and vanishing outside a set of finite measure. Denote by \(\varPhi _f\) such a class of simple functions and observe that

$$\begin{aligned} \int _Efd\mu =\sup _{\varphi \in \varPhi _f}\int _E\varphi d\mu . \end{aligned}$$

Such a definition, while adequate for the Lebesgue measure in \(\mathbb {R}^N\), is not adequate for a general measure space \(\{X,\mathcal {A},\mu \} \). For example let \(\{X,\mathcal {A},\mu \} \) be the measure space of 3.2 of Chap. 3. Then

$$\begin{aligned} \infty =\int _X 1d\mu =\sup _{\varphi \in \varPhi _1}\, \int _X\varphi d\mu =0. \end{aligned}$$

9c More on the Lebesgue Integral

Let \(\{X,\mathcal {A},\mu \} \) be a measure space with \(\mu (X)=1\) and let \(f:X\rightarrow \mathbb {R}\) be \(\mu \)-measurable. In particular, for all Borel sets \(B\in \mathbb {R}\), the set \(f^{-1}(B)\) is \(\mu \)-measurable. Set

$$\begin{aligned} \mu _f(B)=\mu ([f^{-1}(B)])\quad \text { for all Borel sets }\> B\subset \mathbb {R}. \end{aligned}$$
(9.1c)

 

9.1. :

Distribution Measure of a Measurable Function: Verify that \(\mu _f\) is a measure on \(\mathcal {B}\). Verify that if \(f=\chi _E\) for some \(E\in \mathcal {A}\) , then for a Borel set \(B\subset \mathbb {R}\)

$$\begin{aligned} \mu _f(B)=\chi _B(1)\mu (E)+\chi _B(0)\mu (X-E). \end{aligned}$$
9.2. :

A function \(h:\mathbb {R}\rightarrow \mathbb {R}^*\) is Borel measurable if the preimage of a Borel set in \(\mathbb {R}\) is a Borel set in \(\mathbb {R}\). Let h be Borel measurable and integrable with respect to \(\mu _f\). Prove that h(f) is integrable in \(\{X,\mathcal {A},\mu \} \) and

$$\begin{aligned} \int _{X} h(f) d\mu =\int _{\mathbb {R}} h(t)d\mu _f. \end{aligned}$$
(9.2c)

Hint: Given \(\mu _f\), compute the integral of both sides by assuming first that \(h=\chi _B\) where \(B\subset \mathbb {R}\) is a Borel set. Then in view of (9.1c), the integrals equal \(\mu _f(B)=\mu \{f^{-1}(B)\}\). Next assume that h is nonnegative. Compute both sides of (9.2c) by using the definition (7.1) and show that both sides are equal. The general case follows by linear combinations and limiting processes.

9.3. :

For a \(\mu \)-measurable function f on \(\{X,\mathcal {A},\mu \} \) prove that

$$\begin{aligned} \int _X |f|^p d\mu =\int _0^\infty t^p d\mu _f \end{aligned}$$
9.4. :

Equidistributed Measurable Functions: Two measurable function \(f_1\), \(f_2:X\rightarrow \mathbb {R}\) are equidistributed if

$$\begin{aligned} \mu [f_1^{-1}(B)]=\mu [f_2^{-1}(B)]\quad \text { for every Borel set }\> B\subset \mathbb {R}. \end{aligned}$$

Give an example of equidistributed measurable functions \(f_1\ne f_2\). Prove that if \(f_1\) and \(f_2\) are equi-distributed, then

$$\begin{aligned} \int _X h(f_1) d\mu =\int _X h(f_2)d\mu \quad \text { for all}\, h(\cdot )\, \text {as in}\,\mathbf{9.2}. \end{aligned}$$
9.5. :

Expectation and Variance of a Measurable Function: The expectation E (f) and the variance \(\sigma ^2(f)\) of a measurable function f are defined as

$$\begin{aligned} \begin{aligned} E(f)&=\int _X f d\mu =\int _{\mathbb {R}} t d\mu _f\\ \sigma ^2(f)&=\int _X (f-E(f))^2d\mu =\int _{\mathbb {R}}(t-E(f))^2 d\mu _f. \end{aligned} \end{aligned}$$
(9.3c)

The quantity \(\sigma (f)\) , is the standard deviation of f. Prove that \(\sigma ^2(f)=0\) if and only if there exists \(t\in \mathbb {R}\) such that \(f=1\) a.e. in X and \(t=E(f)\).

9.6. :

Let \(f_1\) and \(f_2\) be equidistributed measurable functions on \(\{X,\mathcal {A},\mu \} \). Then \(\sigma ^2(f_1)=\sigma ^2(f_2)\).

 

10c Convergence Theorems

In the problems 10.1–10.10, \(\{X,\mathcal {A},\mu \} \) is a measure space and \(E\in \mathcal {A}\).  

10.1. :

Let \(f:E\rightarrow \mathbb {R}^*\) be integrable and let \(\{E_n\}\) be a countable collection of measurable, disjoint subsets of E such that \(E=\cup E_n\). Then

$$\begin{aligned} \int _Efd\mu =\mathop {\textstyle {\sum }}\limits \,\int _{E_n}fd\mu . \end{aligned}$$
10.2. :

Let \(\mu \) be the counting measure on the positive rationals \(\{r_1,r_2,\dots \}\), and let

$$\begin{aligned} f_n(r_j)=\left\{ \begin{array}{ll} {\displaystyle \frac{1}{j}} &{} \qquad \text { if }\> j<n\\ {}\\ 0&{}\qquad \text { if }\> j\ge n. \end{array} \right. \end{aligned}$$

The sequence \(\{f_n\}\) converges uniformly to a nonintegrable function.

10.3. :

Let \(\mu \) be a finite measure and let \(\{f_n\}\) be a sequence of integrable functions converging uniformly in X. The limiting function f is integrable and one can pass to the limit under integral.

 

1.1 10.1c Another Version of Dominated Convergence

Theorem 10.1c

Let \(\{f_n\}\) be a sequence of integrable functions in E converging a.e. in E to some f. Assume that there exists a sequence of integrable functions \(\{g_n\}\) converging a.e. in E to an integrable function g and such that

$$\begin{aligned} \lim \int _Eg_n d\mu =\int _Eg d\mu . \end{aligned}$$

Assume moreover that \(|f_n|\le g_n\) a.e. in E for all \(n\in \mathbb {N}\). Then f is integrable and

$$\begin{aligned} \lim \int _Ef_n d\mu =\int _Efd\mu . \end{aligned}$$

 

10.4. :

Let \(\{f_n\}:E\rightarrow \mathbb {R}^*\) be a sequence of integrable functions converging to an integrable function f a.e. in E. Then

$$\begin{aligned} \lim \int _E|f_n-f|d\mu =0\quad \text { if and only if }\quad \lim \int _E|f_n|d\mu =\int _E|f|d\mu . \end{aligned}$$
10.5. :

Let \(\{f_n\}:E\rightarrow \mathbb {R}^*\) be a sequence of measurable functions satisfying

$$\begin{aligned} \mathop {\textstyle {\sum }}\limits \int _E|f_n|d\mu <\infty . \end{aligned}$$

Then \(\mathop {\textstyle {\sum }}\limits f_n\) defines, a.e. in E and integrable function and

$$\begin{aligned} \int _E\mathop {\textstyle {\sum }}\limits f_n d\mu =\mathop {\textstyle {\sum }}\limits \int _Ef_n d\mu . \end{aligned}$$
10.6. :

Let \(\{f_n\}:E\rightarrow \mathbb {R}^*\) be a sequence of measurable functions satisfying \(|f_n|\le |f|\) for an integrable function \(f:E\rightarrow \mathbb {R}\). Then

$$\begin{aligned} \int _E\liminf f_n d\mu \le \liminf \int _Ef_n d\mu \le \limsup \int _Ef_n d\mu \le \int _E\limsup f_n d\mu . \end{aligned}$$
10.7. :

Let E be of finite measure and let \(\{f_n\}:E\rightarrow \mathbb {R}^*\) be a sequence of measurable functions satisfying \(|f_n|\le |g|\) for an integrable function \(g:E\rightarrow \mathbb {R}\). If \(\{f_n\}\rightarrow f\) in measure, then

$$\begin{aligned} \int _Ef d\mu =\lim \int _Ef_n d\mu \quad \text { and }\quad \lim \int _E|f_n-f|d\mu =0. \end{aligned}$$
10.8. :

Let E be of finite measure and let \(\{f_n\}:E\rightarrow \mathbb {R}^*\) be a sequence of nonnegative measurable functions converging in measure to f. Then

$$\begin{aligned} \int _Ef d\mu \le \liminf \int _Ef_n d\mu . \end{aligned}$$
10.9. :

Prove that in the Egorov-Severini Theorem 2.1 the assumption \(\mu (E)<\infty \) can be replaced by \(|f_n|\le g\) for an integrable function \(g:E\rightarrow \mathbb {R}^*\).

10.10. :

Let \(\{f_n\}:E\rightarrow \mathbb {R}^*\) be a nonincreasing sequence of nonnegative, measurable functions. Give an example to show that in general

$$\begin{aligned} \lim \int _Ef_n d\mu \ne \int _E\lim f_n d\mu . \end{aligned}$$

Thus in the Monotone Convergence Theorem 8.1, the monotonicity assumption \(f_n\le f_{n+1}\), cannot be replaced with the monotonicity assumption \(f_n\ge f_{n+1}\).

 

In the problems 10.11–10.16, \(\{X,\mathcal {A},\mu \} \) is \(\mathbb {R}^N\) with the Lebesgue measure.  

10.11. :

Let sequences \(\{f_n\}\) be defined by

$$\begin{aligned} f_n(x)=\left\{ \begin{array}{ll} n\>&{}\text { if }\> x\in [0,\frac{1}{n}]\\ 0&{}\text { otherwise}, \end{array} \right. \qquad \quad f_n=\left\{ \begin{array}{ll} \frac{1}{n}\>&{}\text { for }\>0\le x\le n\\ 0&{}\text { for }\> x> n. \end{array} \right. \end{aligned}$$

In either case

$$\begin{aligned} 1=\lim \int _{\mathbb {R}^+}f_ndx \ne \int _{\mathbb {R}^+}\lim f_n dx=0. \end{aligned}$$
10.12. :

Let \(f:\mathbb {R}\rightarrow \mathbb {R}\) be Lebesgue measurable, nonnegative, and locally bounded. Assume that f is Riemann-integrable on \(\mathbb {R}\). Then f is Lebesgue integrable on \(\mathbb {R}\) and

$$\begin{aligned} \int _{\mathbb {R}}fd\mu =\lim \int _{-n}^n f d\mu . \end{aligned}$$
10.13. :

Let \(\{f_n\}\) be the sequence of nonnegative integrable functions defined on \(\mathbb {R}^N\) by

$$\begin{aligned} f_n(x)=\left\{ \begin{array}{ll} {\displaystyle n^N\,\exp {\left\{ \frac{-1}{1-n^2|x|^2}\right\} }} &{} \qquad \text { if }\> |x|<\frac{1}{n}\\ 0&{}\qquad \text { if }\> |x|\ge \frac{1}{n}. \end{array} \right. \end{aligned}$$

The sequence \(\{f_n\}\) converges to zero a.e. in \(\mathbb {R}^N\) and each \(f_n\) is integrable with uniformly bounded integral. However

$$\begin{aligned} \lim \int _{\mathbb {R}^N}f_n dx\ne \int _{\mathbb {R}^N}\lim f_n dx. \end{aligned}$$
10.14. :

Let \(\{f_n\}\) be the sequence defined in 7.7 of the Complements of Chap. 2. The assumptions of the dominated convergence theorem fail and

$$\begin{aligned} \lim \int _0^1 f_n(x)dx=\frac{1}{2}\quad \text { and }\quad \int _0^1\lim f_n(x)dx=0. \end{aligned}$$
10.15. :

The two sequences \(\{x^n\}\) and \(\{n x^n\}\) converge to zero in (0, 1). For the first one can pass to the limit under integral and for the second one cannot.

10.16. :

Integrability and Boundedness: There exist positive, Lebesgue integrable functions on \(\mathbb {R}\), that are infinity at all the rationals. Let \(\{r_n\}\) be the sequence of the rationals in \(\mathbb {R}^+\) and set

$$\begin{aligned} f(x)=\mathop {\textstyle {\sum }}\limits \frac{1}{2^{n}}\frac{e^{-x}}{\sqrt{|x-r_n|}}. \end{aligned}$$

Prove that f is integrable over \(\mathbb {R}^+\) (Proposition 10.2).

 

11c Absolute Continuity of the Integral

The proof of Theorem 11.1 shows that for an integrable function f, given \(\varepsilon >0\), the corresponding \(\delta \) claimed by the theorem, depends upon \(\varepsilon \) and f.

A collection \(\varPhi \) of integrable functions \(f:E\rightarrow \mathbb {R}^*\) is uniformly integrable , if for all \(\varepsilon >0\) there exists \(\delta =\delta (\varepsilon )>0\) such that the conclusion of Theorem 11.1 holds for all \(f\in \varPhi \), for all measurable sets \(\mathcal {E}\subset E\) such that \(\mu (\mathcal {E})<\delta \) (Vitali [169]).  

11.1. :

If \(\varPhi \) is finite then it is uniformly integrable.

11.2. :

Let \(\{f_n\}\) be a sequence of integrable functions such that \(|f_n|\le g\) for an integrable function g. Then \(\{f_n\}\) is uniformly integrable .

11.3. :

Let E be of finite measure and let \(\{f_n\}\) be a sequence of uniformly integrable functions converging a.e. in E to an integrable function f . Prove that (Hint: Egorov–Severini theorem)

$$\begin{aligned} \lim \int _E|f_n-f|d\mu =0. \end{aligned}$$
(11.1c)

This gives an alternate proof of the Lebesgue dominated convergence theorem when \(\mu (E)<\infty \). Give an example showing that the conclusion is false if E is not of finite measure.

11.4. :

Let \(\{f_n\}\) be a sequence of integrable functions in E, satisfying (11.1c) for an integrable f. Then \(\{f_n\}\) is uniformly integrable .

11.5. :

Show that the following sequences of functions defined in \((-1,1)\), are not uniformly Lebesgue integrable.

$$\begin{aligned} \begin{aligned} f_n(x)&=\sqrt{n}e^{-x^2}{n};\qquad&f_n(x)&= \frac{x}{\sqrt{n}}e^{-x^2}{n};\\ f_n(x)&=\frac{n}{n^2 x^2+1};\qquad&f_n(x)&=\frac{n^2 x}{(n^2 x^2+1)^2}. \end{aligned} \end{aligned}$$
(11.2c)
11.6. :

Let E be of finite measure and let \(\{f_t\}:E\rightarrow \mathbb {R}^*\) be a family of functions uniformly integrable in E and such that \(f_t(x)\) is continuous in \(t\in (0,1)\) for every fixed \(x\in E\). Prove that

$$\begin{aligned} (0,1)\ni t\rightarrow \int _Ef_td\mu \quad \text {is continuous}. \end{aligned}$$

 

12c Product of Measures

1.1 12.1c Product of a Finite Sequence of Measure Spaces

Given a finite sequence of measure spaces \(\{X_j,\mathcal {A}_j,\mu _j\}_{j=1}^n\) for some \(n\in \mathbb {N}\) their product space is constructed by the following steps:  

12.1. :

Measurable n -Rectangles. A measurable n -rectangle is a set of the form

$$\begin{aligned} E={\mathop {\textstyle {\prod }}\limits }_{j=1}^nE_j\quad \text { for }\quad E_j\in \mathcal {A}_j\quad \text { for all }\> j=1,\dots ,n. \end{aligned}$$

Denote by \(\mathcal {R}_o^n\) be the collection of all measurable n-rectangles and prove that it forms a semi-algebra.

12.2. :

Measuring Measurable Rectangles. On \(\mathcal {R}_o^n\) introduce the set function

$$\begin{aligned} \mathcal {R}_o^n\ni E\rightarrow \lambda _n(E)=\mathop {\textstyle {\prod }}\limits _{j=1}^n\mu _j(E_j). \end{aligned}$$
(12.1c)

Prove that \(\lambda _n\) is countably additive on \(\mathcal {R}_o^n\). For this use a n-dimensional version of Proposition 12.1. As a consequence \(\lambda _n\) is well defined on \(\mathcal {R}_o^n\) in the sense that, for \(Q\in \mathcal {R}_o^n\), the value \(\lambda _n(Q)\) is independent of a particular partition of elements in \(\mathcal {R}_o^n\) making up Q.

12.3. :

Constructing the Product Measure Space. The set function \(\lambda _n\) on \(\mathcal {R}_o^n\) generates an outer measure \(\mu _e^n\) on \({\mathop {\textstyle {\prod }}\limits }_{j=1}^n X_j\). The latter in turn generates the measure space \(\{Y_n,\mathcal {B}_n,\nu _n\}\), where

$$\begin{aligned} Y_n=\mathop {\textstyle {\prod }}\limits _{j=1}^n X_j,\quad \mathcal {B}_n=\Big (\mathop {\textstyle {\prod }}\limits _{j=1}^n\mathcal {A}_j\Big ), \quad \nu _n=\Big (\mathop {\textstyle {\prod }}\limits _{j=1}^n\mu _j\Big ). \end{aligned}$$

Moreover \(\mathcal {R}_o^n\subset \mathcal {B}_n\) and \(\nu _n(E)=\lambda _n(E)\) for all \(E\in \mathcal {R}_o^n\).

 

1.1.1 12.1.1c Alternate Constructions

Fix an integer \(1\le m<n\) and construct first, by the indicated procedure, the two measure spaces \(\{Y_m,\mathcal {B}_m,\nu _m\}\) and \(\{Y^m,\mathcal {B}^m,\nu ^m\}\), where

$$\begin{aligned} \begin{array}{ccc} {\displaystyle Y_m=\mathop {\textstyle {\prod }}\limits _{j=1}^m X_j},\quad &{}{\displaystyle \mathcal {B}_m=\Big (\mathop {\textstyle {\prod }}\limits _{j=1}^m\mathcal {A}_j\Big )},\quad &{}{\displaystyle \nu _m=\Big (\mathop {\textstyle {\prod }}\limits _{j=1}^m\mu _j\Big )}\\ {}\\ {\displaystyle Y^m=\mathop {\textstyle {\prod }}\limits _{j=m+1}^n X_j}, \quad &{}{\displaystyle \mathcal {B}^m=\Big (\mathop {\textstyle {\prod }}\limits _{j=m+1}^n\mathcal {A}_j\Big )},\quad &{}{\displaystyle \nu ^m=\Big (\mathop {\textstyle {\prod }}\limits _{j=m+1}^n\mu _j\Big )}. \end{array} \end{aligned}$$

Then construct their product

$$\begin{aligned} \{Y_m\times Y^m, (\mathcal {B}_m\times \mathcal {B}^m), (\nu _m\times \nu ^m)\}. \end{aligned}$$
(12.2c)

Denote by \(\mathcal {A}_o^n\) the smallest \(\sigma \)-algebra containing \(\mathcal {R}_o^n\).  

12.4. :

Prove that \(\mathcal {A}_o^n\) is contained in \((\mathcal {B}_m\times \mathcal {B}^m)\) for all \(1\le m<n\).

12.5. :

Prove that for all \(1\le m<n\), the restriction of \((\nu _m\times \nu ^m)\) to \(\mathcal {R}_o^n\) coincides with the function \(\lambda _n\) introduced in (12.1c).

12.6. :

Prove that for all \(1\le m<n\), the product space in (12.2c) is generated by the same outer measure \(\mu _e^n\), generated by \(\lambda _n\) on \(\mathcal {R}_o^n\). Conclude that

$$\begin{aligned} \{Y_n,\mathcal {B}_n,\nu _n\}=\{Y_m\times Y^m,(\mathcal {B}_m\times \mathcal {B}^m), (\nu _m\times \nu ^m)\} \end{aligned}$$
(12.3c)

and thus the finite product of measure spaces is associative.

12.7. :

Prove that the Lebesgue measure in \(\mathbb {R}^N\) coincides with the product of N copies of the Lebesgue measure in \(\mathbb {R}\).

 

13c On the Structure of \((\mathcal {A}\times \mathcal {B}) \)

 

13.1. :

Let \(\{X,\mathcal {A},\mu \} \) and \(\{Y,\mathcal {B},\nu \}\) be complete measure spaces. Let \(A\subset X\) be \(\mu \)-measurable and let \(B\subset Y\) be not \(\nu \)-measurable. Denote by \((\mathcal {A}\times \mathcal {B}) _e\) the outer measure on \(\mathcal {A}\times \mathcal {B}\) that generates the product measure space \(\{X\times Y,(\mathcal {A}\times \mathcal {B}) ,(\mu \times \nu )\}\). Prove that:  

i. :

If \((\mathcal {A}\times \mathcal {B}) _e(A\times B)=0\) then \(A\times B\) is \((\mathcal {A}\times \mathcal {B}) \)-measurable.

ii. :

If \(0<(\mathcal {A}\times \mathcal {B}) _e(A\times B)<\infty \) then \(A\times B\) is not \((\mathcal {A}\times \mathcal {B}) \)-measurable. If \((\mathcal {A}\times \mathcal {B}) _e(A\times B)=\infty \) then \(A\times B\) might be \((\mu \times \nu )\)-measurable. Give an example or an argument.

iii. :

If \(\{X,\mathcal {A},\mu \} \) and \(\{Y,\mathcal {B},\nu \}\) are \(\sigma \)-finite and \((\mathcal {A}\times \mathcal {B}) _e(A\times B)>0\), then then \(A\times B\) is not \((\mathcal {A}\times \mathcal {B}) \)-measurable.

 

13.2. :

Let \(\{X,\mathcal {A},\mu \} \) be non complete and let \(A\subset X\) be non \(\mu \)-measurable, but included in a \(\mu \)-measurable set \(A^\prime \) of measure zero. For every \(\nu \)-measurable set \(B\subset Y\), the rectangle \(A\times B\) is \((\mu \times \nu )\)-measurable. As a consequence, the assumption that both \(\{X,\mathcal {A},\mu \} \) and \(\{Y,\mathcal {B},\nu \}\) be complete, cannot be removed from Proposition 13.4.

13.3. :

Let \(E\subset [0,1]\) be the Vitali non measurable set. The diagonal set \(\mathcal {E}=\{(x,x)|x\in E\}\) is Lebesgue measurable in \(\mathbb {R}^2\) and has measure zero. The rectangle \(E\times E\) is not Lebesgue measurable in \(\mathbb {R}^2\).

13.4. :

(Sierpinski [146]). Let \(\mathbb {R}\) be well ordered by \(\prec \), denote by \(\varOmega \) the first uncountable and let

$$\begin{aligned} X=Y=E_\varOmega =\{x\in \mathbb {R}|x\prec \varOmega \}. \end{aligned}$$

Let \(\mathcal {A}\) be the \(\sigma \)-algebra of the subsets of X that are either countable or their complement is countable. For \(E\in \mathcal {A}\), let \(\mu (E)=0\) if E is countable and \(\mu (E)=1\) otherwise. Consider the set

$$\begin{aligned} E=\{(x,y)\in X\times X\bigm | x\prec y\}. \end{aligned}$$

All the x and y sections of E are measurable. However E is not \((\mu \times \mu )\)-measurable. Indeed if E were measurable, it would have finite measure, since

$$\begin{aligned} E\subset X\times X \quad \text { and }\quad (\mu \times \mu )(X\times X))=\mu (X)\mu (X)=1. \end{aligned}$$

Therefore, it would have to satisfy Proposition 13.4. However

$$\begin{aligned} \int _Y\mu (E_y)d\nu =0\qquad \text { and }\qquad \int _X\mu (E_x)d\mu =1. \end{aligned}$$

 

1.1 13.1c Sections and Their Measure

Given a finite sequence \(\{X_j,\mathcal {A}_,\mu _j\}_{j=1}^n\) of measure spaces, let \(\{Y_n,\mathcal {B}_n,\nu _n\}\) be their product measure space as constructed either in (12.1c)–(12.3c).

For a set \(E\subset Y_n\) and \(y_m\in Y_m\), and \(y^m\in Y^m\) for some \(1\le m<n\), the \(y_m\)-section \(E_{y_m}\) of E and the \(y^m\)-section \(E_{y^m}\) of E are defined as

$$\begin{aligned} \begin{aligned} Y^m\ni E_{y_m}&=\{y^m\bigm |(y_m,y^m)\in E\> \text { for a fixed }\> y_m\in Y_m\}\\ Y_m\ni E_{y^m}&=\{y_m\bigm |(y_m,y^m)\in E\> \text { for a fixed }\> y^m\in Y^m\}. \end{aligned} \end{aligned}$$
(13.4c)

 

13.5. :

Prove that for all \(E\in \mathcal {A}_o^n\) and all \(1\le m<n\) the sections \(E_{y_m}\in \mathcal {B}^m\) and \(E_{y^m}\in \mathcal {B}_m\).

13.6. :

Prove that for all \(E\in \mathcal {A}_o^n\) of finite \(\nu _n\) measure and for all \(1\le m<n\)

$$\begin{aligned} \nu _n(E)=\int _{Y_n}\chi _E d\nu _n =\int _{Y_m}\nu ^m(E_{y_m}) d\nu _m =\int _{Y^m}\nu _m(E_{y^m}) d\nu ^m. \end{aligned}$$
(13.5c)
13.7. :

State and prove a version of Fubini’s Theorem when all the measure spaces \(\{X_j,\mathcal {A}_j,\mu _j\}\), for \(j=1,\dots ,n\), are complete.

13.8. :

State and prove a version of Tonelli’s Theorem when all the measure spaces \(\{X_j,\mathcal {A}_j,\mu _j\}\), for \(j=1,\dots ,n\), are complete and \(\sigma \)-finite.

 

14c The Theorem of Fubini–Tonelli

 

14.1. :

Let \(\omega _N\) be the measure of the unit sphere in \(\mathbb {R}^N\). Then

$$\begin{aligned} \omega _{N+1}=2\omega _N\int _0^{\pi /2} (\sin t)^{N-1}dt. \end{aligned}$$
14.2. :

By the Fubini–Tonelli theorem

$$\begin{aligned} \int _{\mathbb {R}^N}e^{-|x|^2}dx=\mathop {\textstyle {\prod }}\limits _{i=1}^N \int _{\mathbb {R}} e^{-x_i^2}dx_i=\pi ^{N/2}. \end{aligned}$$
14.3. :

Let \(\{X,\mathcal {A},\mu \} \) be [0, 1] with the Lebesgue measure and let \(\{Y,\mathcal {B},\nu \}\) be the rationals in [0, 1] with the counting measure. The function \(f(x,y)=x\) is integrable on \(\{X,\mathcal {A},\mu \} \) and not integrable on the product space.

14.4. :

Let [0, 1] be equipped with the Lebesgue measure \(\mu \) and the counting measure \(\nu \) both acting on the same \(\sigma \)-algebra of the Lebesgue measurable subsets of [0, 1]. The corresponding product measure space is not \(\sigma \)-finite since \(\nu \) is not \(\sigma \)-finite. The diagonal set \(E=\{x=y\}\) is of the type of \(\mathcal {R}_{\sigma \delta }\), and hence \((\mu \times \nu )\)-measurable and

$$\begin{aligned} \nu (E_x)=1\quad \forall \,x\in [0,1]\quad \text { and }\quad \mu (E_y)=0\quad \forall \,y\in [0,1]. \end{aligned}$$

Therefore,

$$\begin{aligned} \int _{[0,1]}\nu (E_x)d\mu =1 \quad \text { and }\quad \int _{[0,1]}\mu (E_y)d\nu =0. \end{aligned}$$

Moreover

$$\begin{aligned} \iint _{[0,1]\times [0,1]}\chi _{E}d(\mu \times \nu )=\infty . \end{aligned}$$
14.5. :

Let f and g be integrable functions on complete measure spaces \(\{X,\mathcal {A},\mu \} \) and \(\{Y,\mathcal {B},\nu \}\), respectively. Then \(F(x,y)=f(x)g(y)\) is integrable on the product space \((X\times Y)\).

14.6. :

Let \(\{X,\mathcal {A},\mu \} \) be any measure space and let \(\{Y,\mathcal {B},\nu \}\) be \(\mathbb {N}\) with the counting measure. Prove that the conclusion of Fubini’s theorem continues to hold even i \(\{X,\mathcal {A},\mu \} \) is not complete, and that the conclusion of Tonelli’s theorem continues to hold even if \(\{X,\mathcal {A},\mu \} \) is neither complete nor \(\sigma \)-finite.

14.7. :

Let \(\{X,\mathcal {A},\mu \} \) and \(\{Y,\mathcal {B},\nu \}\) be both \(\mathbb {N}\) with the counting measure and consider the function

$$\begin{aligned} f(m,n)=\left\{ \begin{array}{rl} 1 &{} \quad \text { if }\> m=n\\ -1 &{} \quad \text { if }\> m=n+1\\ 0 &{} \quad \text { otherwise}. \end{array}\right. \end{aligned}$$

Prove that f is not integrable in the product space \(\{X\times Y,(\mathcal {A}\times \mathcal {B}) ,(\mu \times \nu )\}\) and the conclusion of Fubini’s theorem fails.

14.8. :

Let \(\{X,\mathcal {A},\mu \} \) and \(\{Y,\mathcal {B},\nu \}\) be both (0, 1) with the Lebesgue measure. Let \(f:(0,1)\rightarrow \mathbb {R}\) be measurable and assume that

$$\begin{aligned} (0,1)\times (0,1)\ni (x,y)\rightarrow f(x)-f(y) \end{aligned}$$

is integrable in the product space \(\{X\times Y,(\mathcal {A}\times \mathcal {B}) ,(\mu \times \nu )\}\). Prove that f is integrable in (0, 1).

 

15c Some Applications of the Fubini–Tonelli Theorem

1.1 15.1c Integral of a Function as the “Area Under the Graph”

Let \(\{X,\mathcal {A},\mu \} \) be complete, and \(\sigma \)-finite, and let \(f:X\rightarrow \mathbb {R}^*\) be a nonnegative and integrable. The graph of f on E is the set of points \(\{x:(x,f(x)\}\) and the set of points “under the graph” is

$$\begin{aligned} U_f=\{(x,y)\bigm | x\in X:\> 0\le y<f(x)\}. \end{aligned}$$

Prove that \(U_f\) is measurable in the product measure space of \(\{X,\mathcal {A},\mu \} \) and \(\mathbb {R}\) with the Lebesgue measure dx, and

$$\begin{aligned} \{\text {measure of}\, U_f\}=\int _X f(x)d\mu . \end{aligned}$$

Prove that the graph of f as a subset of \(X\times \mathbb {R}\) is measurable in the product measure and has measure zero.

1.2 15.2c Distribution Functions

Let \(\{X,\mathcal {A},\mu \} \) be a measure space and let \(f:X\rightarrow \mathbb {R}^*\) and \(\{f_n\}:X\rightarrow \mathbb {R}^*\) be nonnegative and integrable. Prove:

Proposition 15.1c

Assume that \(\mu ([f>t])\not \equiv \infty \). Then

$$\begin{aligned}&\lim _{\varepsilon \rightarrow 0}\mu ([f>t+\varepsilon ])= \mu ([f>t])\\&\lim _{\varepsilon \rightarrow 0}\mu ([f>t-\varepsilon ])= \mu ([f\ge t]). \end{aligned}$$

Therefore, the distribution function \(t\rightarrow \mu ([f>t])\) is right-continuous, and it is continuous at a point t if only if \(\mu ([f=t])=0\).

Proposition 15.2c

Let \(\{f_n\}\rightarrow f\) in measure. Then for every \(\varepsilon >0\)

$$\begin{aligned}&\limsup \mu ([f_n>t]) \le \mu ([f>t-\varepsilon ])\\&\liminf \mu ([f_n>t]) \ge \mu ([f>t+\varepsilon ]). \end{aligned}$$

Therefore, \(\mu ([f_n>t])\rightarrow \mu ([f>t])\) at those t where the distribution function of f is continuous.

 

15.1. :

For a measurable function f on \(\{X,\mathcal {A},\mu \} \) set

$$\begin{aligned} \mathbb {R}^*\ni t\rightarrow f_*(t)=\mu \big ([f\le t]\big ) \end{aligned}$$
(15.1c)

prove that \(f_*\) is nondecreasing, right-continuous, and \(f_*(\infty )=\mu (X)\).

15.2. :

Give an example of f and \(\{X,\mathcal {A},\mu \} \) for which \(f_X\) is right-continuous and not continuous.

15.3. :

The function \(f_*\) generates the Lebesgue–Stiltjies measure \(\mu _{f_*}\) on \(\mathbb {R}\). Let \(\mu _f\) be the measure defined by (9.1c). Prove that \(\mu _{f_*}=\mu _f\) on the Borel sets of \(\mathbb {R}\) and that \(\mu _{f_*}\) is the completion of \(\mu _f\).

 

17c The Radon-Nikodým Theorem

 

17.1. :

Let \(\nu \) be the Lebesgue measure in [0, 1] and let \(\mu \) be the counting measure on the same \(\sigma \)-algebra of the Lebesgue measurable subsets of [0, 1]. Then \(\nu \) is absolutely continuous with respect to \(\mu \), but it does not exist a nonnegative, \(\mu \)-measurable function \(f:[0,1]\rightarrow \mathbb {R}^*\) for which \(\nu \) can be represented as in (17.1).

17.2. :

In [0, 1] let \(\mathcal {A}\) be the \(\sigma \)-algebra of the sets that are, either countable or have countable complement. Let \(\mu \) be the counting measure on \(\mathcal {A}\) and let \(\nu :\mathcal {A}\rightarrow \mathbb {R}^*\) be defined by \(\nu (E)=0\) if E is countable and \(\nu (E)=1\) otherwise. Then \(\nu \) is absolutely continuous with respect to \(\mu \) but it does not have a Radon-Nikodým derivative .

17.3. :

Changing the Variables of Integration: Let the assumptions of the Radon-Nikodým theorem hold. If g is \(\nu \)-integrable, \(g(d\nu /d\mu )\) is \(\mu \)-integrable and for every measurable set E

$$\begin{aligned} \int _Eg d\nu =\int _Eg \frac{d\nu }{d\mu }d\mu . \end{aligned}$$
(17.1c)
17.4. :

Linearity of the Radon-Nikodým Derivative: Let \(\mu \) and \(\nu _1\) and \(\nu _2\) be \(\sigma \)-finite measures defined on the same \(\sigma \)-algebra \(\mathcal {A}\). If \(\nu _1\) and \(\nu _2\) are absolutely continuous with respect to \(\mu \)

$$\begin{aligned} \frac{d(\nu _1+\nu _2)}{d\mu }=\frac{d\nu _1}{d\mu }+ \frac{d\nu _2}{d\mu }\qquad \mu \text {-a.e.}. \end{aligned}$$
(17.2c)
17.5. :

The Chain Rule: Let \(\mu \), \(\nu \), and \(\eta \) be \(\sigma \)-finite measures on X defined on the same \(\sigma \)-algebra \(\mathcal {A}\). Assume that \(\mu \) is absolutely continuous with respect to \(\eta \) and that \(\nu \) is absolutely continuous with respect to \(\mu \). Then

$$\begin{aligned} \frac{d\nu }{d\eta }=\frac{d\nu }{d\mu }\frac{d\mu }{d\eta } \quad \text { a.e. with respect to}\, \eta . \end{aligned}$$
(17.3c)
17.6. :

Derivative of the Inverse: Let \(\mu \) and \(\nu \) be two not identically zero, \(\sigma \)-finite measures defined on the same \(\sigma \)-algebra \(\mathcal {A}\) and mutually absolutely continuous. Then

$$\begin{aligned} \frac{d\nu }{d\mu }\ne 0\quad \text { and }\quad \frac{d\mu }{d\nu }=\left( \frac{d\nu }{d\mu }\right) ^{-1}. \end{aligned}$$
(17.4c)

Hint: For all nonnegative \(\nu \)-measurable functions g

$$\begin{aligned} \int _Eg d\nu =\int _Eg\frac{d\nu }{d\mu }d\mu . \end{aligned}$$

This is true for simple functions and hence for nonnegative \(\nu \)-measurable functions g. Now for all \(E\in \mathcal {A}\) of finite \(\mu \)-measure

$$\begin{aligned} \mu (E)=\int _E1 d\mu =\int _E\frac{d\mu }{d\nu }d\nu =\int _E\frac{d\mu }{d\nu }\frac{d\nu }{d\mu }d\mu . \end{aligned}$$

The inverse formula (17.4c) follows from the uniqueness of the Radon-Nikodým derivative.

17.7. :

Let \(\{X,\mathcal {A},\mu \} \) be [0, 1] with the Lebesgue measure. Let \(\{E_n\}\) be a measurable partition of [0, 1] and let \(\{\alpha _n\}\) be a sequence of positive numbers such that \(\mathop {\textstyle {\sum }}\limits \alpha _n<\infty \). Find the Radon-Nikodým derivative of the measure

$$\begin{aligned} \mathcal {A}\ni E\longrightarrow \nu (E)=\mathop {\textstyle {\sum }}\limits \,\alpha _n\mu (E\cap E_n) \end{aligned}$$

with respect to the Lebesgue measure.

 

Proposition 17.1c

Let \(\mu \) and \(\nu \) be two measures defined on the same \(\sigma \)-algebra \(\mathcal {A}\). Assume that \(\nu \) is finite. Then \(\nu \) is absolutely continuous with respect to \(\mu \) if and only if for every \(\varepsilon >0\) there exists \(\delta >0\) such that \(\nu (E)<\varepsilon \) for every set \(E\in \mathcal {A}\) such that \(\mu (E)<\delta \).

Proof

(Necessity) If not, there exist \(\varepsilon >0\) and a sequence of measurable sets \(\{E_n\}\), such that \(\nu (E_n)\ge \varepsilon \) and \(\mu (E_n)\le 2^{-n}\). Let \(E=\limsup E_n\) and compute

$$\begin{aligned} \nu (E)=\nu (\limsup E_n)\ge \limsup \nu (E_n)\ge \varepsilon . \end{aligned}$$

On the other hand for all \(n\in \mathbb {N}\)

$$\begin{aligned} \mu (E)\le \mu \big (\mathop {\textstyle {\bigcup }}\limits _{j=n}^\infty \,E_j\big )\le \mathop {\textstyle {\sum }}\limits _{j=n}^\infty \,\mu (E_j)\le \frac{1}{2^{n-1}}. \end{aligned}$$

\(\blacksquare \)

 

17.8. :

The proposition might fail if \(\nu \) is not finite. Let \(X=\mathbb {N}\) and for every subset \(E\subset \mathbb {N}\), set

$$\begin{aligned} \mu (E)=\mathop {\textstyle {\sum }}\limits _{n\in E}\,\frac{1}{2^n}\qquad \nu (E)=\mathop {\textstyle {\sum }}\limits _{n\in E}\,2^n. \end{aligned}$$
17.9. :

More on \(\varvec{\nu \ll \mu }\) : There exist \(\sigma \)-finite measures \(\nu \) on the Lebesgue measurable sets of \(\mathbb {R}\), absolutely continuous with respect to the Lebesgue measure \(\mu \) on \(\mathbb {R}\) and such that \(\nu (E)=\infty \) for every measurable set E with nonempty interior. Let \(\{r_n\}\) be the sequence of the rationals in \(\mathbb {R}\) and set

$$\begin{aligned} g(x)=\mathop {\textstyle {\sum }}\limits \frac{1}{2^{n}}\frac{e^{-|x|}}{|x-r_n|}\quad \text { and } \quad \nu (E)=\int _Egd\mu \end{aligned}$$

for all Lebesgue measurable set \(E\subset \mathbb {R}\).

17.10. :

Let \(\{X,\mathcal {A},\mu \} \) be a measure space and let \(\nu \) be a signed measure on \(\mathcal {A}\) of finite total variation \(|\nu |\). Then \(\nu \ll \mu \) if and only if for every \(\varepsilon >0\) there exists \(\delta >0\) such that \(|\nu |(E)<\varepsilon \) for all \(E\in \mathcal {A}\) such that \(\mu (E)<\delta \). Give an counterexample if \(|\nu |\) is not finite.

 

18c A Proof of the Radon-Nikodým Theorem When Both \(\mu \) and \(\nu \) are \(\sigma \)-Finite

A more constructive proof of Theorem 17.1 can be given if both \(\mu \) and \(\nu \) are \(\sigma \)-finite. Assume first that both \(\mu \) and \(\nu \) are finite. Let \(\varPhi \) be the family of all measurable nonnegative functions \(\varphi :X\rightarrow \mathbb {R}^*\) such that

$$\begin{aligned} \int _E\varphi d\mu \le \nu (E)\quad \text { for all } \> E\in \mathcal {A}. \end{aligned}$$

Since \(0\in \varPhi \) such a class is not empty. For two given functions \(\varphi _1\) and \(\varphi _2\) in \(\varPhi \) the function \(\max \{\varphi _1;\varphi _2\}\) is in \(\varPhi \). Indeed for any \(E\in \mathcal {A}\)

$$\begin{aligned} \int _E\max \{\varphi _1;\varphi _2\}d\mu&= \int _{E\cap [\varphi _1\ge \varphi _2]}\varphi _1d\mu + \int _{E\cap [\varphi _2>\varphi _1]}\varphi _2d\mu \\&\le \nu (E\cap [\varphi _1\ge \varphi _2])+ \nu (E\cap [\varphi _2<\varphi _1])=\nu (E). \end{aligned}$$

Since \(\nu \) is finite

$$\begin{aligned} M=\sup _{\varphi \in \varPhi }\int _X\varphi d\mu \le \nu (X)<\infty . \end{aligned}$$

Let \(\{\varphi _n\}\) be a sequence of functions in \(\varPhi \) such that

$$\begin{aligned} \lim \int _X \varphi _nd\mu =M \end{aligned}$$

and set \(f_n=\max \{\varphi _1,\cdots ,\varphi _n\}\). The sequence \(\{f_n\}\) is nondecreasing, and \(\mu \)-a.e. convergent to a function f that belongs to \(\varPhi \). Indeed by monotone convergence, for every measurable set E

$$\begin{aligned} \int _Efd\mu =\lim \int _Ef_nd\mu \le \nu (E). \end{aligned}$$

Such a limiting function is the f claimed by the theorem. For this it suffices to establish that the measure

$$\begin{aligned} \mathcal {A}\ni E\longrightarrow \eta (E) =\nu (E)-\int _Efd\mu \end{aligned}$$

is identically zero. If not, there exists a set \(A\in \mathcal {A}\) such that \(\eta (A)>0\). Since both \(\nu \) and \(\eta \) are absolutely continuous with respect to \(\mu \), for such a set, \(\mu (A)>0\). Also, since \(\mu \) is finite, there exists \(\varepsilon >0\) such that

$$\begin{aligned} \xi (A)=\eta (A)-\varepsilon \mu (A)>0. \end{aligned}$$

The set function

$$\begin{aligned} \mathcal {A}\ni E\longrightarrow \xi (E)=\eta (E)-\varepsilon \mu (E) \end{aligned}$$

is a signed measure on \(\mathcal {A}\). Therefore, by Proposition 16.1 the set A contains a positive subset \(A_o\) of positive measure. In particular

$$\begin{aligned} \eta (E\cap A_o)-\varepsilon \mu (E\cap A_o) \ge 0\quad \text { for all }\> E\in \mathcal {A}. \end{aligned}$$

From this and the definition of \(\eta \)

$$\begin{aligned} \varepsilon \mu (E\cap A_o)\le \nu (E\cap A_o)- \int _{E\cap A_o}fd\mu \quad \text { for all }\> E\in \mathcal {A}. \end{aligned}$$

The function \((f+\varepsilon \chi _{A_o})\) belongs to \(\varPhi \). Indeed, for every measurable set E

$$\begin{aligned} \int _E(f+\varepsilon \chi _{A_o})d\mu&=\int _{E-A_o}fd\mu + \int _{E\cap A_o}(f+\varepsilon )d\mu \\&\le \nu (E-A_o)+\nu (E\cap A_o)=\nu (E). \end{aligned}$$

This however contradicts the definition of M since

$$\begin{aligned} \int _X(f+\varepsilon \chi _{A_o})d\mu =M+\varepsilon \mu (A_o)>M. \end{aligned}$$

If \(g:X\rightarrow \mathbb {R}^*\) is another nonnegative measurable function by which the measure \(\nu \) can be represented, let \(A_n=[f-g\ge \frac{1}{n}]\). Then for all \(n\in \mathbb {N}\)

$$\begin{aligned} \frac{1}{n}\mu (A_n)\le \int _{A_n}(f-g)d\mu = \nu (A_n)-\nu (A_n)=0. \end{aligned}$$

Thus \(f=g\) \(\mu \)-a.e. in X.

Assume next that \(\mu \) is \(\sigma \)-finite and \(\nu \) is finite. Let \(E_n\) be a sequence of measurable, expanding sets such that

$$\begin{aligned} \mu (E_n)\le \mu (E_{n+1})<\infty \quad \text { and }\quad X=\mathop {\textstyle {\bigcup }}\limits E_n \end{aligned}$$

and denote by \(\mu _n\) the restriction of \(\mu \) to \(E_n\). Let \(f_n\) be the unique function claimed by the Radon-Nikodým theorem for the pair of finite measures \(\{\mu _n;\nu \}\). By construction \(f_{n+1}\bigm |_{E_n}= f_n\) for all n. The function f claimed by the theorem is \(f=\sup f_n\). Indeed if \(E\in \mathcal {A}\), by monotone convergence

$$\begin{aligned} \nu (E)=\lim \nu (E\cap E_n)= \lim \int _Ef_nd\mu =\int _Efd\mu . \end{aligned}$$

The uniqueness of such a f is proved as in the case of \(\mu \) finite. Finally a similar argument, establishes the theorem when also \(\nu \) is \(\sigma \)-finite.\(\blacksquare \)

Rights and permissions

Reprints and permissions

Copyright information

© 2016 Springer Science+Business Media New York

About this chapter

Cite this chapter

DiBenedetto, E. (2016). The Lebesgue Integral. In: Real Analysis. Birkhäuser Advanced Texts Basler Lehrbücher. Birkhäuser, New York, NY. https://doi.org/10.1007/978-1-4939-4005-9_4

Download citation

Publish with us

Policies and ethics